Dirac Delta Function (x+a)

In summary, an expert summarizer of content provided the following summary: -An equation for the result of integrating a Dirac Delta Function over a region including x=a and zero otherwise is -If x=a, the first term is 1, while the second term is undefined. -For x>-1, the second term becomes 1/2.
  • #1
Pual Black
92
1

Homework Statement


hi
i have to find the result of
##\int_{0}^{\pi}[\delta(cos\theta-1)+ \delta(cos\theta+1)]sin\theta d\theta##

Homework Equations


i know from Dirac Delta Function that
##\int \delta(x-a)dx=1##
if the region includes x=a and zero otherwise

The Attempt at a Solution


first i make the substitution
let ##\cos\theta=x##
then ##\sin\theta d\theta=-dx##
at ##\theta=0 \rightarrow x=1##
at ##\theta=\pi \rightarrow x=-1##

##\int_{1}^{-1}[\delta(x-1)+ \delta(x+1)](-dx)##

##\int_{-1}^{1}[\delta(x-1)+ \delta(x+1)](dx)##

##\int_{-1}^{1}\delta(x-1)dx+ \int_{-1}^{1}\delta(x+1)(dx)##

so the first term is equal one but what about the second term. I know that the result must be 2
 
Physics news on Phys.org
  • #2
In ##\delta(x-a) = \delta(x+1)##, what is the value of ##a##?
 
  • #3
DrClaude said:
In ##\delta(x-a) = \delta(x+1)##, what is the value of ##a##?
In L.H.S
X must be equal a. Else dirac delta will be zero.
R.H.S
X=-1
Or a=-1
Maybe I didn't understand your question.
 
  • #4
Pual Black said:
a=-1
Correct. So what is the value of the corresponding integral?
 
  • #5
Pual Black said:
##\int_{-1}^{1}\delta(x-1)dx+ \int_{-1}^{1}\delta(x+1)(dx)##

so the first term is equal one

Why? I think that the first term is undefined. Something strange is happening at ##x = 1##, so write it as the limit,
$$\int_{-1}^{1} \delta \left( x -1 \right) dx = \lim_{ a \to 1} \int_{-1}^{a} \delta \left( x -1 \right) dx.$$
This two-sided limit, however, does not exist, because
$$\lim_{ a \to 1^-} \int_{-1}^{a} \delta \left( x -1 \right) dx = 0,$$
and
$$\lim_{ a \to 1^+} \int_{-1}^{a} \delta \left( x -1 \right) dx = 1.$$
In the first one-sided limit, ##x = 1## is not in the region of integration, while in the second one-sided limit, ##x = 1## is in the region of integration. Since the two one-sided limits are not equal, the two-sided limit does not exist.

I suppose that you mean the second one-sided limit, i.e., that
$$\int_{-1}^{1} \delta \left( x -1 \right) dx = \lim_{ a \to 1^+} \int_{-1}^{a} \delta \left( x -1 \right) dx.$$
 
  • #6
DrClaude said:
Correct. So what is the value of the corresponding integral?
Since a=-1 is in the range of integral. The integral is equal one?
 
  • #7
Pual Black said:

Homework Statement


hi
i have to find the result of
##\int_{0}^{\pi}[\delta(cos\theta-1)+ \delta(cos\theta+1)]sin\theta d\theta##

Homework Equations


i know from Dirac Delta Function that
##\int \delta(x-a)dx=1##
if the region includes x=a and zero otherwise

The Attempt at a Solution


first i make the substitution
let ##\cos\theta=x##
then ##\sin\theta d\theta=-dx##
at ##\theta=0 \rightarrow x=1##
at ##\theta=\pi \rightarrow x=-1##

##\int_{1}^{-1}[\delta(x-1)+ \delta(x+1)](-dx)##

##\int_{-1}^{1}[\delta(x-1)+ \delta(x+1)](dx)##

##\int_{-1}^{1}\delta(x-1)dx+ \int_{-1}^{1}\delta(x+1)(dx)##

so the first term is equal one but what about the second term. I know that the result must be 2

How do you know the result must be 2? You are integrating through only half of a ##\delta## function. Some people would say that the result is not defined at all, others would say the final result should be 1 (not the 2 that you give). We could also argue that for ##a > -1## we have ##\int_{-1}^a \delta(x+1) \, dx = 0.15865## or ##= 0.84134## (for both of which there exist perfectly good arguments/justifications). However, I would personally take ##\int_{-1}^a \delta(x+1) \, dx = 1/2##, for more-or-less good reasons.
 
Last edited:
  • #8
Ray Vickson said:
How do you know the result must be 2? You are integrating through only half of a ##\delta## function. Some people would say that the result is not defined at all, others would say the final result should be 1 (not the 2 that you give). We could also argue that for ##a > -1## we have ##\int_{-1}^a \delta(x+1) \, dx = 0.15865## or ##= 0.84134## (for both of which there exist perfectly good arguments/justifications). However, I would personally take ##\int_{-1}^a \delta(x+1) \, dx = 1##, for more-or-less good reasons.

The reason why i think the result is 2 because of a problem in Jackson Electrodynamic.
I have now uploaded a pdf file and at page 1 you can see that the integral of dirac delta should be 2.
 

Attachments

  • hw05a.pdf
    123.6 KB · Views: 253

1. What is the definition of the Dirac Delta Function (x+a)?

The Dirac Delta Function (x+a) is a mathematical function that is zero everywhere except at x = -a, where it is infinite. It is often used to represent a point charge or impulse in physics.

2. How is the Dirac Delta Function (x+a) different from a regular function?

The Dirac Delta Function (x+a) is not a regular function because it is not defined at all points. It is considered a generalized function or distribution, and its value at x = -a is not defined in a traditional sense, but rather as a limit of other functions.

3. What is the purpose of using the Dirac Delta Function (x+a)?

The Dirac Delta Function (x+a) is often used in physics and engineering to model point sources of energy or mass, as well as to simplify mathematical equations and calculations. It is also useful in signal processing and quantum mechanics.

4. Can the Dirac Delta Function (x+a) be integrated?

Yes, the Dirac Delta Function (x+a) can be integrated, but it requires the use of advanced mathematical techniques such as the Riemann-Stieltjes integral. Its integral from negative infinity to positive infinity is equal to 1.

5. What are some real-life examples of the Dirac Delta Function (x+a)?

The Dirac Delta Function (x+a) is commonly used in physics to represent point charges, such as an electron, or point masses, such as a planet. It is also used in signal processing to model a sudden change in a signal, and in quantum mechanics to represent an instantaneous measurement of a particle's position or momentum.

Similar threads

  • Calculus and Beyond Homework Help
Replies
31
Views
2K
  • Calculus and Beyond Homework Help
Replies
3
Views
2K
  • Calculus and Beyond Homework Help
Replies
7
Views
1K
  • Calculus and Beyond Homework Help
Replies
13
Views
486
  • Calculus and Beyond Homework Help
Replies
5
Views
687
  • Calculus and Beyond Homework Help
Replies
8
Views
236
  • Calculus and Beyond Homework Help
Replies
7
Views
285
  • Calculus and Beyond Homework Help
Replies
7
Views
1K
  • Calculus and Beyond Homework Help
Replies
3
Views
563
  • Calculus and Beyond Homework Help
Replies
3
Views
799
Back
Top